LSAT and Law School Admissions Forum

Get expert LSAT preparation and law school admissions advice from PowerScore Test Preparation.

 Administrator
PowerScore Staff
  • PowerScore Staff
  • Posts: 8917
  • Joined: Feb 02, 2011
|
#23837
Complete Question Explanation

Resolve the Paradox. The correct answer choice is (B)

The paradox in this stimulus involves rent-control, a concept intended to protect tenants from sharp increases, actually can be detrimental: the policy can create incentives for landlords to avoid both maintenance and new building.

The question stem asks for the choice which best explains the reluctance of landlords to maintain or build.

Answer choice (A): Since un-rent-controlled apartments are not discussed in the stimulus, this choice is irrelevant. Regardless of their preferences, the relevant tenants live in rent-controlled apartments, so the landlord doesn’t necessarily have the option to providing accommodations that are not rent-controlled.

Answer choice (B): This is the correct answer choice. While the rent controls are intended to help the renters, the controls can, as this answer choice provides, make it more difficult to earn a return on one’s investment in building or maintaining. This explains the reluctance referenced in the stimulus.

Answer choice (C): This does not explain why rent-controlled landlords would be reluctant to build or maintain, so this answer choice cannot be correct.

Answer choice (D): How rent control is implemented and removed is irrelevant to the paradox presented in the stimulus, which deals only with apartments that are rent-controlled.

Answer choice (E): Like answer choice (A), this choice presents an irrelevant choice between two options are not both available. Thus, tenant preference in this context is not relevant, and does not explain landlords’ reluctance to build or maintain their buildings.
 whardy21
  • Posts: 48
  • Joined: Sep 30, 2018
|
#65204
My answer choice was A. I thought A explains both sides of the paradox. I don't see how B addresses the part of the paradox about rent control helps tenants in the short run. Please explain. Thanks.
 Adam Tyson
PowerScore Staff
  • PowerScore Staff
  • Posts: 5153
  • Joined: Apr 14, 2011
|
#65350
"Helping tenants" was not part of what the question stem asked you to explain, whardy21, and so it is not relevant to our analysis of the answers. The stem was explicitly about the reluctance of landlords to maintain their units or build more, so that is all we need to explain. It's not really a paradox that we are trying to resolve, but just something we need to better understand. We classify this as a "Resolve the Paradox" question simply because that is the best fit for any question that asks us to explain something is not fully explained by the stimulus. It's more of a "mystery" than it is a "paradox", but the same basic idea applies in how to approach the answers. Look for new info in the answers that makes you react by saying to yourself "oh, okay, now I get why that happened."

Try approaching this as a Strengthen question, where you are going to strengthen the position taken by the landlords, and answer B should become much more attractive to you! You'll also see why answer A tells us nothing about why landlords don't maintain their existing units, and even less about why they aren't building more.

Get the most out of your LSAT Prep Plus subscription.

Analyze and track your performance with our Testing and Analytics Package.